University Calculus: Early Transcendentals (3rd Edition)

Published by Pearson
ISBN 10: 0321999584
ISBN 13: 978-0-32199-958-0

Chapter 7 - Section 7.3 - Hyperbolic Functions - Exercises - Page 418: 46

Answer

$$\sqrt 3 \ln | \sinh (\dfrac{\theta}{\sqrt 3})| +C= \sqrt 3 \ln |e^{\theta/\sqrt{3}}-e^{-\theta/\sqrt{3}}| +C$$

Work Step by Step

Given: $\int \coth (\dfrac{\theta}{\sqrt 3}) d\theta$ Plug in $\dfrac{\theta}{\sqrt 3}=t$ and $d \theta =\sqrt 3 dt$ Thus, $\int \coth (\dfrac{\theta}{\sqrt 3}) d\theta=\sqrt 3 \int \coth t dt=\sqrt 3 \int \dfrac{\cosh t}{\sinh t} dt$ Now, plug in $\sinh t =u \implies \cosh t dt =du$ Thus, we have: $=\sqrt 3 \int \dfrac{du}{u}$ or, $= \sqrt 3 \ln |u| +C$ $= \sqrt 3 \ln | \sinh (\dfrac{\theta}{\sqrt 3})| +C$ $= \sqrt 3 \ln | \frac{e^{\theta/\sqrt{3}}-e^{-\theta/\sqrt{3}}}{2}| +C$ $= \sqrt 3 \ln |e^{\theta/\sqrt{3}}-e^{-\theta/\sqrt{3}}| +C$
Update this answer!

You can help us out by revising, improving and updating this answer.

Update this answer

After you claim an answer you’ll have 24 hours to send in a draft. An editor will review the submission and either publish your submission or provide feedback.